PT51.S4.G3 - a locally known guitarist's demo

Henry AnHenry An Alum Member
edited September 2017 in Logic Games 123 karma

Hi all,

I have a question on how I am supposed to read a certain rule in sequencing games. The fifth rule in this game states: Each rock classic is immediately preceded on the CD by a new composition. I wrote it out as [ R -> NR ]
Looking at 7sage's explanation for this question, this rule limits the first spot to only N, because if you put R in the first spot you would be forcing an N to a zero spot, which is a contradiction.

I am actually quite puzzled by this rule because in an earlier PT (PT20 S3 Game 3), such mechanics did not apply. Rule 1 of this game states: If a purple bead is adjacent to a yellow bead, any bead that immediately follows and any bead that immediately precedes that pair must be red. I wrote this rule out as [ (PY or YP) -> R (YP or PY) R ]. Then similarly wouldn't YP or PY not able to go to the first and second spots, as this would force a R to a zero spot? But clearly, Question 13 allows P to be placed at first and Y to be second.

Why do these similar rules force out different restrictions? Is there something I am missing? Thanks in advance!

Comments

  • LSATcantwinLSATcantwin Alum Member Sage
    edited September 2017 13286 karma

    In the bead game the statement is IF YP or PY then R has to go in front. Both Y and P can still go first as long as they aren't next to each other.

    For example YRP is fine and PPR is fine. This rule only talks about when PY are next to each other.

    In the other game a NR MUST come before R. So R will never go first because a NR has to come right before it.

  • Henry AnHenry An Alum Member
    123 karma

    @LSATcantwin Thanks for your reply! I'm sorry I wasn't clear in my question. In Question 13 in the bead game, PY are indeed next to each other, yet the correct answer allows P to be in the first spot and Y in the second, yet no R before the pair. I'll have to edit my post to accommodate this.

  • LSATcantwinLSATcantwin Alum Member Sage
    13286 karma

    @"Henry An" said:
    @LSATcantwin Thanks for your reply! I'm sorry I wasn't clear in my question. In Question 13 in the bead game, PY are indeed next to each other, yet the correct answer allows P to be in the first spot and Y in the second, yet no R before the pair. I'll have to edit my post to accommodate this.

    Give me a second. I'll look into both games again. I was on mobile before and am now at work on a PC. I'll try and flush out an answer.

  • LSATcantwinLSATcantwin Alum Member Sage
    13286 karma

    Ah so I didn't read the rule all the way. This is sort of tricky. The rule is saying that if PY then if anything comes before it, it must be red and if anything comes after it, it must be red. This allows for nothing to come before it. It still hinges on the if, we are never told that it MUST happen. Just that if the situation occurs, and there is something before it or after it, we have to use red.

  • Henry AnHenry An Alum Member
    123 karma

    @LSATcantwin ah! so if I'm understanding correctly, in the bead game there is an "if" statement within an "if" statement, which allows the second "if" statement to not happen. Thank you for your explanation!

Sign In or Register to comment.